The lines shown below are parallel. If the green line has a slope of -17, what is the slope of the red line? ​

Answers

Answer 1

Answer:

it would be 17

Step-by-step explanation:


Related Questions

In the figure below, K lies between J and L. Find the location of K so that JK is 1/7 of JL.​

Answers

Answer:

-22 is the value of K.

Step-by-step explanation:

A simple way to see how far a mid-point or end-point is from another is by adding or subtracting. For this figure I added to see how many numbers would add twice to get from -15 to -29. -14 is the exact number from -15 to -29; divide that by 2 to get half of -14 which is -7. -7 + (-15) gives you -22.

The location of K so that JK is 1/7 of JL is -22.

Given that, K lies between J and L.

We need to find the location of K so that JK is 1/7 of JL.​

What is a line segment?

In geometry, a line segment is a part of a line that is bounded by two distinct end points and contains every point on the line that is between its endpoints.

Now, the distance between the points J and L on the number line is 14.

So, 1/7(14)=2

A simple way to see how far a mid-point or end-point is from another is by adding or subtracting. For this figure, I added to see how many numbers would add twice to get from -15 to -29. -14 is the exact number from -15 to -29; divide that by 2 to get half of -14 which is -7. -7 + (-15) gives you -22.

Therefore, the location of K so that JK is 1/7 of JL is -22.

To learn more about the line segment visit:

https://brainly.com/question/25727583.

#SPJ2

The area of the square base is S(x), where x is the length of the concrete foundation, and the area of the base of the cylindrical tank is C ( r ), r is the radius of the cylindrical tank. What does the expression S ( C ( r ) ) represent?

Answers

Answer:

C(r) is the area of the base of a cylindrical tank with radius r.

We know that this area is:

C(r) = pi*r^2

S(x) is the area of the square base where x is the side length of the base.

S(x) = x*x

Now, we have the expression:

S( C(r)) wich means that we are evaluating the function S(x) in the point x = C(r)

Now, we have a problem here.

In S(x), S(x) is in units of area, and x is in units of length.

And the same for C(r), C(r) is in units of area, and r in units of length.

then we can not have x = C(r)

Then this expression does not represent anything, because the units do not match, so it has no physical sense.

The current value of a baseball card is 50 times its original value v (in dollars). The baseball card is worth $62. Write and solve an equation to find the original value of the baseball card.

Answers

Answer:

62*50=3100

enjoy

Please help me idk what to do I have other ones too

Answers

Answer:

B

Step-by-step explanation:

all the other ones are either 50 - 12 or 50 + 12, but B is the total amount of money (x) minus 50($) is 12($).

not sure if that makes sense.

Answer:

b. Haley withdrew $50 from her savings account . After withdrawal she had only $12. How much money  was in  the account before withdrawal

Step-by-step explanation:

What set of reflections would carry triangle ABC onto itself?

Answers

I think I could be letter C

Find the midpoint of the two points (-4,6) & (1.4)​

Answers

Answer:

(-3/2, 5)

Step-by-step explanation:

(x₁ + x₂) / 2 , (y₁ + y₂) / 2

(-4 + 1) / 2 , (6 + 4) / 2

-3/2 , 10/2

(-3/2, 5)

-3-(-8)-(-2)=Xggggggggggggggggggggggg

Answers

Answer:

x is 7

Step-by-step explanation:

-3-(-8)-(-2) = x

             7 = x

Point M is the midpoint of segment AC. Point M is located at (25.5, 60.1), and point C is located at
(18.3, 72.5).

M (25.5, 60.1)

C (18.R 72.5)
Find the location of point A. Round to the nearest tenth.

Answers

Answer:255.601

Step-by-step explanation:

Tasha was $422.75 last week. Of that paycheck $42.75 was extra overtime pay for 3 hours. If the rest of the check was Tasha’s weekly earnings for 40 hours, what is her weekly per-hour rate?

Answers

$9.50


Explanation:

You subtract the total of her check minus the amount she worked for overtime & then divide the total from that by 40 & you’ll get $9.50.

The solution is $ 9.50

The weekly per hour rate of Tasha is $ 9.50

What is an Equation?

Equations are mathematical statements with two algebraic expressions flanking the equals (=) sign on either side.

It demonstrates the equality of the relationship between the expressions printed on the left and right sides.

Coefficients, variables, operators, constants, terms, expressions, and the equal to sign are some of the components of an equation. The "=" sign and terms on both sides must always be present when writing an equation.

Given data ,

Let the equation be represented as A

Now , the value of A is

The weekly per hour of Tasha last week = $ 422.75

The extra overtime pay of Tasha for 3 hours = $ 42.75

The total number of hours working = 40 hours

So , the equation will be

The weekly per hour of Tasha without overtime = weekly per hour of Tasha last week - overtime pay of Tasha for 3 hours

Substituting the values in the equation , we get

The weekly per hour rate of Tasha without overtime = 422.75 - 42.75

The weekly per hour of Tasha without overtime = $ 380

So , The weekly per hour rate of Tasha A = The weekly per hour rate of Tasha without overtime / total number of hours working

Substituting the values in the equation , we get

The weekly per hour rate of Tasha A = 380 / 40

The weekly per hour rate of Tasha A = $ 9.50

Therefore , the value of A is $ 9.50

Hence , The weekly per hour rate of Tasha is $ 9.50

To learn more about equations click :

https://brainly.com/question/19297665

#SPJ5

Ron asked 18 classmates whether they prefer granola bars over muffins. he used a calculator to compare the number of classmates who said yes to the total number he surveyed. the calculator showed the result as 0.66666667. how many students prefer granola bars over muffins?

Answers

Answer: 12

Explanation:

Multiply 0.66666667 with 18 to get

0.66666667*18 = 12.00000006

The 6 at the very end is due to rounding error. In reality the 6's in 0.66666667 should go on forever. That 7 is from rounding up.

Anyways, the result 12.00000006 should be 12 exactly.

Note how

12/18 = 0.66666667

when using a calculator.

Answer:

12

Step-by-step explanation:

Use equilvalent fractions 0.6 repeating is 2/3 than convert it over 18 students which is equal to 12/18 students chose granola bars.

w−2+2w=6+5w how do I solve this I have been stuck on this for like 10 minutes.

Answers

Answer:

W= -3

Step-by-step explanation:

3w=6+5w

-5w on each side

-2w=6

Divide by - 2 on each side

W=-3

QR is a radius of r and PQ is tangent to r. Find the value of x. Please help me!! I am in dire need! What is X?! Thank you for any assistance. :)

Answers

Answer:

  54

Step-by-step explanation:

The Pythagorean theorem applies.

  (x +36)^2 = 72^2 + x^2

  x^2 +72x +1296 = x^2 +5184

  72x = 3888 . . . . . . subtract x^2+1296

  x = 54 . . . . . . . . . . . divide by 72

A laundry detergent recommends 1/4 cup of detergent for a 1/2 load of clothes. How much detergent is recommended for 4 loads of clothes?

Answers

Answer:

2 cups

Step-by-step explanation:

Because you have 4 loads of clothes you would have to use

1/4 of a cup 8 times

1/4*8=8/4 simplified is 2 which no fractions

so the answer is 2 cups for 4 loads of clothes.

Evaluate 20x y given that x=15 and y= 10.​

Answers

Answer:

3000

Step-by-step explanation:

20xy20×15×10just let the x and y into it..

A phone call costs $0.58 per minute for the first three minutes and $0.15 per minute for each additional minute. If the total charge for the call was $4.29, how many minutes long was the call Explain the answer in terms of the problem. Show all work.

Answers

Answer:

20 minutes

Step-by-step explanation:

.58 times 3 equal 1.74

4.29 - 1.74 = 2.55

2.55 divided by .15 = 17

17 + 3 = 20

hopefully this helps you :)

pls mark brainlest ;)

okay last one but
5(n^2 + n) – 3(n^2 – 2n)?

Answers

Answer =2n^2+11n

you welcome:)

Answer:

2n² + 11n

Step-by-step explanation:

5(n² + n) – 3(n²2 – 2n) =

= 5n² + 5n - 3n² + 6n

= 2n² + 11n

Please help I will be very grateful for this

Answers

Answer:

____________

the answer is B

___________

explanation

Factor out the greatest common factor from each group.

Group the first two terms and the last two terms.

(x^2 - 2x) - 3xy + 6y

Factor out the greatest common factor (GCF) from each group.

x(x - 2) - 3y (x - 2)

Factor the polynomial by factoring out the greatest common factor, x - 2

(x - 2) (x - 3y)

Answer:

Step-by-step explanation:

your answer would be B.

add and subtract the second turn to the expression and Factor by grouping

Solve the equation 3/4=m+1/4

Answers

Answer:

1/2 = m

Step-by-step explanation:

3/4=m+1/4

Subtract 1/4 from each side

3/4 - 1/4=m+1/4-1/4

2/4 = m

Simplify

1/2 = m

Answer:

m=  

2

1

​  

=0.500

Step-by-step explanation:

Which choice correctly explains which product is the better buy? Coffee Brand A is sold for $6.99 per pound Coffee Brand B is sold for $5.69 for 12 ounces Question 1 options: A Coffee Brand A is a better buy because its unit rate is $2.29 per ounce B Coffee Brand A is a better buy because its unit rate is $0.44 per ounce C Coffee Brand B is a better buy because its unit rate is $2.11 per ounce D Coffee Brand B is a better buy because its unit rate is $0.47 per ounce.

Answers

Answer: Choice B

Coffee Brand A is a better buy because its unit rate is $0.44 per ounce

=======================================================

Explanation:

Brand A is sold for $6.99 per pound, which is the same as saying $6.99 per 16 ounces. The unit rate is 6.99/16 = 0.436875 = 0.44 dollars per ounce

Brand B is sold for $5.69 for 12 ounces. The unit rate is  5.69/12 = 0.47416 = 0.47 dollars per ounce

Brand A is cheaper per ounce compared to brand B.

------------------

Here's another way we can look at it

Brand A is $6.99 per 16 ounces. We can set up the equation below

6.99 dollars = 16 ounces

then divide both sides by 16 to turn "16 ounces" into "1 ounce"

So,

6.99 dollars = 16 ounces

6.99/16 dollars = 16/16 ounces

0.436875 dollars = 1 ounce

0.44 dollars = 1 ounce

is the unit rate for brand A.

Similar steps would happen with brand B as well.

Could someone please help? I literally can’t figure this out

Answers

Answer:

56

Step-by-step explanation:

(x - 9(a/8)) = a a = 16

Answers

Answer:

x = 34

Step-by-step explanation:

(x - 9(16/8)) = 16

(x - 9(2)) = 16                        Divide 16 and 8

x - 18 = 16                             Multiply 9 and 2

x = 34                                   Add 18 to both sides




if 6 orange cost nu.48what is the cost of each​

Answers

Step-by-step explanation:

Hey there!!

The cost of 6 orange = nu.48

The cost of 1 orange = nu.48/6

= nu.8

Therefore, the cost of each orange is nu.8.

Hope it helps...

0.48/6=0.08, so the cost of each orange is $0.08.

Is 0.7 a rational or irrational number? why is a rational number

Answers

Answer:

0.7 is a rational number.

Explanation:

Because it is a terminating decimal expansion.

Answer:

The decimal 0.7 is a rational number. It is read as seven tenths and is equivalent to the fraction 7/10. Since it can be written as a fraction

12 people can comfortably fit in a 5 x 5 foot square. Use this value to estimate the size of a crowd that is 8 feet deep on both sides of the street along a 2 mile section of a parade route.

Answers

Answer:

40,550

Step-by-step explanation:

Given that 12 people can comfortably fit in a 5 × 5 foot square

Area of square = 5²= 25 ft²

The dimension Given by : 8 feets depth on both sides along a 2 mile section signifies a rectangular area.

Area of rectangle = (length × width)

Recall :

1 mile = 5280 feets

2 miles = 5280 × 2 = 10,560 feets

Area = 8 × 10560 = 84480 sq feets

84480 / 25 = 3379.2

Estimated number of people will be :

3379.2 × 12 = 40,550.4

Hence, estimated number of people = 40,550

Dividing circumference with radius

Answers

If you divide the circumference C over the radius r, you get 2pi

2pi = C/r

this is because

C = 2pi*r

is one formula to find the circumference. Another formula is

C = pi*d

which works because d = 2r, ie the diameter is twice the radius.

(x+2)^3/4=27. Solve for x.

Answers

Answer:

x = 79.

Step-by-step explanation:

Simplify both sides of the equation, then isolate the variable.

20 points Without solving the systems, determine if there is 0, 1 or infinite solutions. Explain how you know.

Answers

Answer:

Infinite Solutions.

Step-by-step explanation:

3x+2y=8 for x:

3x+2y=8

3x+2y+−2y=8+−2y(Add -2y to both sides)

3x=−2y+8 3x 3 = −2y+8 3 (Divide both sides by 3)

x= −2 3 y+ 8 3

Substitute −2/3y + 8/3 for x in 4.5x+3y=12:

4.5x+3y=12

4.5( −2/3y+ 8/3 )+3y=12

12=12(Simplify both sides of the equation)

12+−12=12+−12(Add -12 to both sides)

0=0

Infinitely many solutions.

Answer:

Infinite Solutions.

Step-by-step explanation:

3x+2y=8 for x:

3x+2y=8

3x+2y+−2y=8+−2y(Add -2y to both sides)

3x=−2y+8 3x 3 = −2y+8 3 (Divide both sides by 3)

x= −2 3 y+ 8 3

Substitute −2/3y + 8/3 for x in 4.5x+3y=12:

4.5x+3y=12

4.5( −2/3y+ 8/3 )+3y=12

12=12(Simplify both sides of the equation)

12+−12=12+−12(Add -12 to both sides)

0=0

Infinitely many solutions.

Patrick and his friends Jerome and marina collect football cards. Altogether they have 3000 cards. Patrick has 1533 cards in his collection and Jerome has 837 cards. Which statements about the football cards are correct ?

Answers

Answer:

i'm not sure what the statements are but here is some information

p + j + m = 3000

1533 + 837 + m = 3000

m = 630

Marina has 630 cards

I need help on this problem ​

Answers

Answer:

The answer is c.

Answer:

I hope the information below helps

Step-by-step explanation:

Option 1

[tex]\left|-0.2\right| = 0.2\\\\\mathrm{Apply\:absolute\:rule}:\quad \:|-a|=a,\:\quad \:a>0\\\\0.2 =0.2\\\\True[/tex]

Option 2

[tex]-\left|0.2\right| = 0.2\\\mathrm{Apply\:absolute\:rule}:\quad \:|a|=a,\:\quad \:a\ge 0\\\\-0.2 =0.2\\False[/tex]

Option 3

[tex]\left|0.2\right| =0.2\\\mathrm{Apply\:absolute\:rule}:\quad \left|a\right|=a,\:a\ge 0\\\\0.2 =0.2[/tex]

Option 4

[tex]-\left|0.2\right| =- 0.2\\\mathrm{Apply\:absolute\:rule}:\quad \:|a|=a,\:\quad \:a\ge 0\\\\-0.2 =-0.2\\True[/tex]

is this correct? And if i’m not can you explain please?

Answers

Answer:

Option C

Step-by-step explanation:

[tex] {9x}^{10} \times ( - {12}^{13} )[/tex]

[tex] = > {( - 12 \times 9)}x^{10 + 13} [/tex]

[tex] = > { - 108x}^{23} [/tex]

You’re right the answer is C
Other Questions
How can I evaluate this when x=1/2 and y= -5 A diner was eating at a restaurant. In a plate of noodles was a rather large shard of broken, off-white ceramic. Upon finding this physical hazard, the diner showed the problem to server and requested a replacement plate of noodles. What should the manager of the restaurant do to make sure this situation doesn't happen again? -This is for Culinary Art! How do they get the power in the Republic ALFA Electronics has been trading around 70. A customer tells his registered representative that if 1,000 shares of the stock can be purchased in a single attempt, the customer will take it. If not, the customer is not interested and the order should be canceled immediately. How should the representative enter this order why are Basic Map Types so important?? 1. A car travels at 55 km/h for 6.0 hours. How far does it travel? 2. Simplify the expression6xy*(2xy = x?) What is the main purpose of this article?Select...What is it?Since its founding in 1956, the Duke of Edinburgh Award (DofE) scheme has inspired people from all backgrounds to achieve the bronze, silver and gold awards.Who is it for?As long as youre aged between 14 and 24, you can complete a Duke of Edinburgh Award. Participants get a discount card, so the costs of the expedition are more affordable for everyone.Why do it?There are benefits to achieving an award, particularly the gold level. It builds your confidence, helps you keep fit, and shows employers and colleges that you can work hard to meet a difficult challenge. Someone plans to float a small, totally absorbing sphere 0.518 m above an isotropic point source of light, so that the upward radiation force from the light matches the downward gravitational force on the sphere. The sphere's density is 22.2 g/cm3, and its radius is 2.12 mm. (a) What power would be required of the light source If the patient is less than 70 years old, the loading dose is 80 Howell Units2 per kilogram of DM. One Howell Unit is equivalent to 0.0020 mg of pure heparin. 2. Calculate how many mg of pure heparin is in 80 Howell Units. Do not worry about the kilograms of DM yet. 00.1600mg The loading dose, in mg of heparin for a 5-year-old with a total body weight of 40 lbs., with 10% bodyfat (that is, 90% of that 40 lbs. is lean body mass). You may also be interested to know that 1 kg = 2.20 lbs. Think about the steps you need to go through to find the correct dose. Remember, first you need to find the dosing mass.... Rachel plans to study 2 hours for her next exam how might plotting her The definition of terrorism fluctuates according to the interests of the group defining the term.A. TrueB. False What is the center of the circle that has a diameter whose endpoints are (9, 7) and (-3, -5)? Why is this statement considered a theme and not a plot summary?Success is possible through hard work and perseverance.A. It establishes a conflict that will lead to a resolutionB. It states universal truth about lifeC. It introduces the opinion of an author D. It gives the chronological order of events in a story Determine how many terms should be used to estimate the sum of the entire series with an error of less than 0.001. Write an equation through: (-1, 2)and (0, -1) To qualify for the long jump competition in track, an athlete must jump a combined distance of more than 16 meters in 3 jumps. Juan jumped 4.76 meters, 5.428 meters, and 5.9 meters. Did Juan qualify for (meet) the competition 1. Raphael pays Better Buy $800 to for a new high-definition television (HDTV) and its installation. He's attracted by Better Buy's guarantee that he'll be happy with the new HDTV, or he'll get his money back. 2. Better Buy pays Firedog $650 to install the HDTV 3. Firedog buys hardware worth $50 from The Home Station Compute contributions to GDP, using the expenditure approach. Assume that The Home Station receives the hardware at no charge and that other costs are zero Hint: Add the amount of money spent by buyers of final goods and services Which of the following would be included in the expenditure method of calculating GDP? 1. Raphael spends $800 2. Firedog spends $50 3. Better Buy spends $650 The total contribution to GDP, measured by the expenditure method, is_________ Now use the following table to compute contributions to GDP, employing the resource cost-income approach. In particular, indicate the costs of intermediate goods and the value added at each stage of production Stage of Production The Home Station Firedog Better Buy Sale Value $50 $650 $800 Cost of Intermediate Goods Resource Cost-Income sale value value added cost of intermediate goods The contribution to GDP that you found using the expenditure approach corresponds to the sum of the ______ at each stage of production. Can someone send the golden number ? Which of the following is an example of a primary source?O A biography about George WashingtonO Atextbook about George WashingtonO An autobiography about George WashingtonO A documentary about George Washington